1answer.
Ask question
Login Signup
Ask question
All categories
  • English
  • Mathematics
  • Social Studies
  • Business
  • History
  • Health
  • Geography
  • Biology
  • Physics
  • Chemistry
  • Computers and Technology
  • Arts
  • World Languages
  • Spanish
  • French
  • German
  • Advanced Placement (AP)
  • SAT
  • Medicine
  • Law
  • Engineering
dolphi86 [110]
3 years ago
5

Como se relacionan los valores de los ochos en 880

Mathematics
1 answer:
velikii [3]3 years ago
5 0

en el número 880, los dos ochos representan 800 y 80

You might be interested in
A box in the shape of a rectangular prism has a volume of 96 cubic feet. The box has a length of (x+8) feet, a width of x feet,
anygoal [31]
96=(x+8)*x*(x-2)=x^3 +6x^2 -16x. Solve to get x= -6,-4,4. Negative distance doesn't make sense, so x=+4. Therefore, length is (x+8)=(4+8)= 12, width=x=4, and height=(x-2)=(4-2)=2.
3 0
3 years ago
The following table shows the estimated populations and annual growth rates for four countries in the year 2000. Find the expect
Liula [17]

Answer:

After 25 years the population will be:

  • Australia: 22271200
  • China: 1580220878
  • Mexico: 157380127
  • Zaire: 112794819

Step-by-step explanation:

Growth rate problem that has a growth rate proportional to the population size can be solved using the equation:

 P(t) = P₀eʳᵗ

  • t is your unit of time. It could be days, or hours, or minutes. It changes depending on each problem. In this problem, t is measured in years because you're jumping from 2000 to 2025. Years just makes the most sense to measure that leap in time.
  • P(t) is the population at time t. An example in this problem could be P(20) would be the population 20 years after the initial count. or maybe P(12) would be the population 12 years after the initial count. or P(0) would be the initial count of the population.
  • P₀ is the initial population at P(0)
  • r is the growth rate.<u><em> Don't forget to convert the percentage to its decimal form</em></u>

Now that everything is set out, lets use the equation to solve for our answer.

P(t) = P₀eʳᵗ

<u>Australia:</u>

P(t)=(19169000)e^{(0.006)t}

after 25 years

P(25)=(19169000)e^{(0.006)(25)}=22271200.6

<u>China:</u>

P(t) = (1261832000)e^{(.009)t}

after 25 years:

P(25)=(1261832000)e^{(.009)(25)}=1580220878

<u>Mexico:</u>

P(t) = (100350000)e^{(0.018)t}

after 25 years:

P(25)=(100350000)e^{(0.018)(25)}=157380127.8

<u>Zaire:</u>

P(t) = (51965000)e^{(0.031)t}

after 25 years:

P(25)=(51965000)e^{(0.031)(25)}=112794819.9

6 0
4 years ago
Hello, Brainly community!
ioda

Answer:

(B)  \displaystyle \frac{W(3.1) - W(2.9)}{0.2}

General Formulas and Concepts:

<u>Calculus</u>

Limits

Derivatives

  • The definition of a derivative is the slope of the tangent line.

Derivative Notation

Instantaneous Rates

  • Tangent Line: \displaystyle f'(x) = \frac{f(b) - f(a)}{b - a}

Step-by-step explanation:

Since we are trying to find a <em>rate</em> at which W(t) changes, we must find the <em>derivative</em> at <em>t</em> = 3.

We are given 2 close answer choices that would have the same <em>numerical</em> answer but different <em>meanings</em>:

  1. (A)  \displaystyle  \lim_{t \to 3} W(t)
  2. (B)  \displaystyle \frac{W(3.1) - W(2.9)}{0.2}

If we look at answer choice (A), we see that our units would simply just be volume. It would not have the units of a rate of change. Yes, it may be the closest numerically correct answer, but it does not tell us the <em>rate</em> at which the volume would be changing and it is not a derivative.

If we look at answer choice (B), we see that our units would be cm³/s, and that is most certainly a rate of change. Answer choice (B) is also a <em>derivative</em> at <em>t</em> = 3, and a derivative tells us what <em>rate</em> something is changing.

∴ Answer choice (B) will give us the best estimate for the value of the instantaneous rate of change of W(t) when <em>t</em> = 3.

Topic: AP Calculus AB/BC (Calculus I/I + II)

Unit: Differentiation

Book: College Calculus 10e

8 0
3 years ago
5(-3x - 2) - (x - 3) = -4 (4x +5) + 13
S_A_V [24]
Start off by distributing the numbers into the parentheses:

5(-3x - 2) - (x - 3) = -4(4x + 5) + 13
-15x - 10 - (x - 3) = -16x - 20 + 13

(Note: It's super important to be careful when opening up negative parentheses! -(x-3) is not just - x - 3, it is actually -x + 3 since the negative is distributed in every number!)

-15x - 10 - x + 3 = -16x - 20 + 13
-16x - 7 = -16x - 7
-16x = -16x
0 = 0

There is an infinite number of solutions in this equation.

(When you get 0=0 when solving for a variable, that means that said variable will have infinite solutions, that is, any number plugged into the equation will work)
4 0
3 years ago
Read 2 more answers
The value, V (t), in dollars, of a stock t months after it is purchased is modeled by the following equation.
11Alexandr11 [23.1K]

Answer:

Step-by-step explanation:V(t)= 42(1-e ^{1.5t} ) +36 is given value in dollars of a stock in t months after it is purchased.

a) Substitute 1 for t

V(1) = 42(1-e^{-1.5} )+36 \\=67.852\\=67.85

V(12) = V(1) = 42(1-e^{-1.5*12} )+36 \\=77.00

b) Find derivative for V

V'(t) = 42(-1.5) e^{-1.5t} )\\\\=-63e^{-1.5t}

c) When V(t) = 75

V(t)=75= 42(1-e ^{1.5t} ) +36\\1-e ^{1.5t=0.9286\\=-2.639

d) As t tends to infinity, exponent being in negative t tends to 0

So V tends to 42(1-0)+36 = 78

3 0
3 years ago
Other questions:
  • a calculator has 50 keys in five colors grey black blue yellow and green their 6 gray keys for every 7 blue Keys write a possibl
    12·2 answers
  • How do i solve and graph y=x^2+5x+6
    5·2 answers
  • What is the product of (-4)(7) and why?
    10·2 answers
  • Why is it important to write numbers in different ways
    6·1 answer
  • Sophie uses 18 beads to make a necklace.Three sixths of the beads are purple.How many beads are purple
    10·2 answers
  • Students were ask to their favorite subject 8 students chose science, 3 times as many students chose reading as science the numb
    14·2 answers
  • The speed - time graph of a car is given here. Using the data in the graph calculate the total distance covered by the car.
    5·1 answer
  • I need help with this please!!
    15·2 answers
  • WILL GIVE BRAINLIEST PLEASE HELP ME
    12·1 answer
  • In the coordinate plane, you are given JKL with J(2, 3), K(10,4), and L(6, 10). Using the distance formula, classify JKL
    10·1 answer
Add answer
Login
Not registered? Fast signup
Signup
Login Signup
Ask question!